mΖΗ - 67
pleas please please help!! i’m doing angles

Answers

Answer 1

Answer:

could u take a picture of the angles please

Step-by-step explanation:


Related Questions

- CA Geometry A Illuminate Credit 4 FF.pdf

Answers

Answer:

hii

Step-by-step explanation:

If a 750 ml bottle of juice costs £1.90 and a 1 litre bottle of the same juice costs £2.50 then the 750 ml bottle is better value.

Answers

Answer:

The 1 liter bottle is better value

Step-by-step explanation:

Cost of 750 ml = £1.90

Cost of 1 liter = £2.50

1000 ml = 1 liter

Cost per 250 ml

750 ml / 3 = £1.90 / 3

250 ml = £0.6333333333333

Approximately,

£ 0.633

Cost per 250 ml

1 liter / 4 = £2.50 / 4

250 ml = £0.625

The 750 ml bottle is not a better value

The 1 liter bottle is better value

What is the period 3 pi and 4 pi

Answers

Answer:

i think i know the answer sorry if im wrong but i would say B

Step-by-step explanation:

Which of the following is a correct tangent ratio for the figure? A) tan (24) 76 B) tan(76°) °= 2 C) tan(76°) = D) tan(8") = 24 76​

Answers

Given question is incorrect; here is the complete question.

"Which of the following is a correct tangent ratio for the figure attached"

A) tan(76°) = [tex]\frac{24}{8}[/tex]

B) tan (76°) = [tex]\frac{8}{24}[/tex]

C) tan (24°) = [tex]\frac{76}{8}[/tex]

D) tan (8°) = [tex]\frac{24}{76}[/tex]

Option A will be the correct option.

   From the figure attached,

Given triangle is a right triangle.Measure of one angle = 76°Measure of two sides of the triangle are 24 and 8units.

By applying tangent ratio of angle having measure 76°.

tan(76°) = [tex]\frac{\text{Opposite side}}{\text{Adjacent side}}[/tex]

             = [tex]\frac{24}{8}[/tex]

    Therefore, Option (A) is the correct option.

Learn more,

https://brainly.com/question/14169279

Please help!

A line intersects the points (-2, 8) and
(4, 12). Find the slope and simplify
completely.
Help Resource
Slope
[?]
= +
Hint: m =
y2-yi
X2-X1
Enter

Answers

Answer:

2/3

Step-by-step explanation:

We can use the slope formula

m = (y2-y1)/(x2-x1)

 = (12-8)/(4 - -2)

  (12-8)/(4+2)

  4/6

   2/3

Determine the period

Answers

Answer:

3 units

Step-by-step explanation:

The period of a wave is the time taken to complete a cycle of motion of the wave

In the given figure, the graduations of the x-axis, which is the usually time axis = 1 unit

At the origin, (0, 0), the vertical displacement of the wave = 0

The maximum value of the wave function is between x = 0 and x = 1

The minimum value of the wave function is between x = 2 and x = 3

At the point (3, 0) the value of the wave function is again 0, and a cycle of the wave is completed

Therefore, the period of the wave = 3 units of the x-variable

convert 4 seconds to hour​

Answers

Answer:

0.00111111 hrs

Step-by-step explanation:

Have a nice day

Answer:

4/3600 = .001111 hr

Step-by-step explanation:

4 seconds * 1 hour            *    1 minute            =    4/3600 = .001111 hr

                    60 minutes          60 seconds

Solve. -7x+1-10x^2=0

Answers

Answer:

[tex]-7x+1-10x^2=0[/tex]

[tex]-10x^2-7x+1=0[/tex]

[tex]quadratic\:equation:-[/tex] [tex]ax^2+bx+c=0[/tex]

[tex]solutions:-\\\\x_{1,\:2}=\frac{-b\pm \sqrt{b^2-4ac}}{2a}[/tex]

[tex]For \\A=-10\\B=-7\\C=1[/tex]

[tex]x_{1,\:2}=\frac{-\left(-7\right)\pm \sqrt{\left(-7\right)^2-4\left(-10\right)\cdot \:1}}{2\left(-10\right)}[/tex]

[tex]\sqrt{\left(-7\right)^2-4\left(-10\right)\cdot \:1}=\sqrt{89}[/tex]

[tex]x_{1,\:2}=\frac{-\left(-7\right)\pm \sqrt{89}}{2\left(-10\right)}[/tex]

[tex]x_1=\frac{-\left(-7\right)+\sqrt{89}}{2\left(-10\right)},\:x_2=\frac{-\left(-7\right)-\sqrt{89}}{2\left(-10\right)}[/tex]

[tex]\frac{-\left(-7\right)+\sqrt{89}}{2\left(-10\right)}=-\frac{7+\sqrt{89}}{20}[/tex]

[tex]\frac{-\left(-7\right)-\sqrt{89}}{2\left(-10\right)}=\frac{\sqrt{89}-7}{20}[/tex]

[tex]x=\frac{\sqrt{89}-7}{20}[/tex]

OAmalOHopeO

On Monday morning at 8:00 a.m. the temperature is – 14 o C. Over the
next 6 hours the temperature rises 6 o C. Between 2:00 p.m. on Monday
and 8:00 a.m. on Tuesday the temperature drops 9 o C. Over the next 6
hours the temperature rises only 4 o C. What is the temperature at 2:00
p.m. on Tuesday?

Answers

Answer : the temperature is -13 degrees Celsius

Perform the indicated operation. Be sure the answer is reduced.

4x/2x+y + 2y/2x+y



4
2
1

Answers

Answer:

2y + y/x + 2

Step-by-step explanation:

is the answer.....

PLS HELP QUESTION ATTACHED

Answers

Answer:

A

Step-by-step explanation:

the -1 represents the graph going down by 1

juans pencil box measures 6 cm long. if the length of the diagonal is 10 cm what is the width of the pencil box

Answers

Answer:

8 cm

Step-by-step explanation:

We can use the Pythagorean theorem to solve since we have a right triangle

a^2 +b^2 = c^2 where a and b are the legs and c is the hypotenuse

a^2 +6^2 = 10^2

a^2 +36 = 100

a^2 = 100-36

a^2 = 64

Taking the square root of each side

sqrt(a^2) = sqrt(64)

a =8

Answer:

8 cm

Step-by-step explanation:

Use the Pythagorean theorem- [tex]a^{2} +b^{2} =c^{2}[/tex]

leg a: 6cm

leg b: unknown

hypotenuse: 10cm

Therefore [tex]6^{2} +x^{2} =10^{2} = 36+x^{2} =100[/tex]

Subtract 36 to 100 to isolate the [tex]x^{2}[/tex].    [tex]x^{2} =64[/tex]

Square root both sides and get your answer of 8cm

Show Workings.
Question is in attached image.​

Answers

Answer:

A.]A chord of a circle of diameter 40 cm subtends an angle of 70° at the centre of the circle.

Solution given;

diameter [d]=40cm

centre angle [C]=70°

(a) Find the perpendicular distance be tween the chord and the centre of the circle.

Answer:

we have

the perpendicular distance be tween the chord and the centre of the circle=[P]let

we have

P=d Sin (C/2)

=40*sin (70/2)

=22.9cm

the perpendicular distance be tween the chord and the centre of the circle is 22.9cm.

(b) Using = 3.142, find the length of the minor arc.

Solution given;

minor arc=[tex]\frac{70}{360}*πd=\frac{7}{36}*3.142*40[/tex]

=24.44cm

the length of the minor arc. is 24.44cm.

B.]In the diagram, XZ is a diameter of the cir cle XYZW, with centre O and radius 15/2 cm.

If XY = 12 cm, find the area of triangle XYZ.

Solution given:

XY=12cm

XO=15/2cm

XZ=2*15/2=15cm

Now

In right angled triangle XOY [inscribed angle on a diameter is 90°]

By using Pythagoras law

h²=p²+b²

XZ²=XY²+YZ²

15²=12²+YZ²

YZ²=15²-12²

YZ=[tex]\sqrt{81}=9cm[/tex]

:.

base=9cm

perpendicular=12cm

Now

Area of triangle XYZ=½*perpendicular*base

=½*12*9=54cm²

the area of triangle XYZ is 54cm².

Answer:

Question 1

a)

d = 40 cm ⇒ r = 20 cm

Let the perpendicular distance is x.

Connecting the center with  the chord we obtain a right triangle with hypotenuse of r and leg x with adjacent angle of 70/2 = 35°.

From the given we get:

x/20 = cos 35°x = 20 cos 35°x = 16.383 cm (rounded)

b)

The minor arc is 70° and r = 20

The length of the arc is:

s = 2πr*70/360° = 2*3.142*20*7/36 = 24.437 cm (rounded)Question 2

Since XZ is diameter, the opposite angle is the right angle, so the triangle XYZ is a right triangle.

r = 15/2 cm ⇒ XZ = d = 2r = 2*15/2 = 15 cm

Find the missing side, using Pythagorean:

[tex]YZ = \sqrt{XZ^2 - XY^2} = \sqrt{15^2-12^2} = \sqrt{81} = 9[/tex]

The area of the triangle:

A = 1/2*XY*YZ = 1/2*12*9 = 54 cm²

Calculate the answer to the correct number of significant figures: (1.705 + 0.5067) / (0.2 * 1.243) = ______.


8.897


8.8966


8.9


9


8.90

Answers

Answer:

8.9

Step-by-step explanation:

they said to the sig. figure so since it's 8.8966, so the answer will be 8.9

The answer to the correct number of significant figures is 8.897, the correct option is A.

What are Significant Figures?

Significant figures is a positional notation, these are the digits that are required to understand the quantity of something.

The expression is

⇒(1.705 + 0.5067) / (0.2 * 1.243)

=2.2117/0.2486

=8.89662

≈ 8.897

To know more about Significant figures

https://brainly.com/question/14359464

#SPJ2

This drawing would be a step in finding which point of concurrency in a triangle?

Answers

Answer:

B. Orthocenter

Step-by-step explanation:

The orthocenter of a triangle is the point of intersection of the three altitudes (the perpendicular from a vertex to the side facing the vertex) of a triangle

The steps used in constructing the orthocenter includes the steps for drawing of the perpendicular to the sides of the triangle from a point which is the vertex opposite the side of the triangle

Therefore, in the given diagram, a perpendicular to the side KS is drawn from the (vertex) point W, by drawing an arc with center at W that intersects the side KS at two points. Rom the points of intersection of the arc drawn from W intersects KS, with the compass, two arcs are drawn to intersect at a point from which a line drawn to W, is perpendicular to KS

Therefore;

The correct option is B. orthocenter

The length of a rectangular-shaped patio is 3 feet shorter than four times its width.
A. Write an equation to represent the area of the patio.
B. If the patio has an area of 370ft square, find the dimensions of the patio

Answers

Answer:

A. A = 4x² – 3x

B. Dimension = 37 ft × 10 ft

Step-by-step explanation:

A. Writing an equation to represent the area of the patio.

From the question given above,

Length (L) = (4x – 3) ft

Width (W) = x ft

Area (A) =?

Area of a rectangle is given by:

Area (A) = Length (L) × Width (W)

A = L × W

A = (4x – 3)x

Clear bracket

A = 4x² – 3x

B. Determination of the dimensions of the patio.

Area (A) = 370 ft²

Dimensions =?

Next, we shall determine the width and length. This can be obtained as follow:

From (A) above,

A = 4x² – 3x .... (1)

A = 370 ....... (2)

Equating equation 1 and 2, we have

4x² – 3x = 370

Rearrange

4x² – 3x – 370 = 0

Solving by formula method:

4x² – 3x – 370 = 0

Coefficient of x² (a) = 4

Coefficient of x (b) = –3

Constant (c) = –370

x = –b ± √(b² – 4ac) / 2a

x = –(–3) ± √(–3² – (4×4×–370)) / 2×4

x = 3 ± √(9 + 5920) / 8

x = 3 ± √(5929) / 8

x = 3 ± 77 / 8

x = (3 + 77) / 8 or (3 – 77) / 8

x = 80/8 or –74/8

x = 10 or –37/4

Since measurement can not be negative, thus,

x = 10

Width (W) = x ft

x = 10

W = 10 ft

Length (L) = 4x – 3 ft

x = 10 ft

L = 4(10) – 3

L = 40 – 3

L = 37 ft

Finally, we shall determine the dimensions. This is illustrated below:

W = 10 ft

L = 37 ft

Dimension =?

Dimension = L × W

Dimension = 37 ft × 10 ft

True or False?
k = 3 over 4 is a solution to the inequality 12k + 2 < 12.
True
False

Answers

Answer:

False.

Step-by-step explanation:

...................

We know that k is 3/4. We can plug in 3/4 into k. 12(3/4) + 2 < 12. This gives us 36/48. 36/48 simplified is 3/4. Our equation is currently 3/4 + 2 < 12. We need common denominators to add 3/4 and 2. 2 with a denominator of 4 is is 8/4. 3/4 + 8/4 is 11/4. 11/4 < 12. 11/4 as a mixed number is 2 3/4. 2 3/4 as a decimal is 2.75. 2.75 < 12. 2.75 is less than 12. This is true.

Evaluate the expression 3(5 + 2)(7 - 2) using order of operations.

Answers

The answer is 36. Do the parentheses first and then multiply by 3

Answer:

105

Step-by-step explanation:

The order of operations is written as PEMDAS. These letters stand for:

-Parentheses

-Exponents

-Multiplication

-Division

-Addition

-Subtraction

We follow these steps in order to solve expressions efficiently. Now, we are going to use PEMDAS to evaluate the expression 3(5+2)(7-2) step by step.

3(7)(5)  The first step is to simplify the numbers in the parentheses.

There are no exponents, so we skip to the next step, multiplication.

(3*7)(5)

21(5)

105

PEMDAS is no longer needed because 105 has come out to be our answer.

I hope this helps you out! Have an an awesome day :3

Help PLEASE eeeeeeeeeeeeeeeee

Answers

Answer:

0.6 is the answer please

which statement must be true about line TU?

Answers

line tu is parallel to rs

Answer:

line TU has no slope in the diagram above

Can I know the answer for the above questions

Answers

Answer:

Step-by-step explanation:

what is 8/9 divide 2/3?

Answers

Answer:

4/3

Step-by-step explanation:

8/9 ÷ 2/3

Simplify the complex fraction.

4/3

Step-by-step explanation:

8/9 ÷ 2/3

Simplify

4/3 is the correct answer

Which statement is true regarding the angles in the figure below?

Triangle E D F. A diagonal line is drawn from point D to form an angle.
Angle D is an exterior angle because it shares a side with the triangle.
Angle D is an exterior angle because it is not inside the triangle.
Angle D is an exterior angle because it is formed by one side of the triangle and by extending another side of the triangle.
Angle D is not an exterior angle.

Answers

An exterior angle is an angle supplementary to one of the interior angles.  

In other words, an exterior angle is the angle between one of the sides, and the extension of an adjacent side.

In the given diagram, the angle D is measured from one of the sides, but not to the extension of an adjacent side.

Therefore angle D is not an exterior angle.

Option D is the correct one!

An exterior angle is an angle supplementary to one of the interior angles.  

We have given that,

Triangle E D F. A diagonal line is drawn from point D to form an angle.

Angle D is an exterior angle because it shares a side with the triangle.

Angle D is an exterior angle because it is not inside the triangle.

Angle D is an exterior angle because it is formed by one side of the triangle and by extending another side of the triangle.

Angle D is not an exterior angle.

What is the exterior angle?

an exterior angle is an angle between one of the sides and the extension of an adjacent side.

In the given diagram, the angle D is measured from one of the sides, but not to the extension of an adjacent side.

Therefore angle D is not an exterior angle.

To learn more about the exterior angle visit:

https://brainly.com/question/24242466

#SPJ5

A parallelogram is cut out of a 12-inch by 8-inch sheet of paper. There are four right triangle remnants. Two have the dimensions 2 inches by 9 inches, and the other two have the dimensions 3 inches by 6 inches. A parallelogram is shown. An altitude is drawn from one point to the opposite side to form a right angle. The length of the base is 9.22 inches. The resulting parallelogram has a base of approximately 9.22 inches.

Answers

Answer:

96

36

60

6.51

Step-by-step explanation:

Answer:96, 36, 60, 6.51

Step-by-step explanation:

Which of the following is the solution to the equation x + 8.6 = 36.2?
10.4
17.2
27.6
44.8

Answers

Answer:

27.6

Step-by-step explanation:

x+8.6=36.2

Subtract 8.6 from both sides.

x=36.2-8.6

Subtract 8.6 from 36.2 to get 27.6.

x=27.6

The letter is c so 27.6

x+8.6=36.2 ( original problem)

Subtract 8.6 from both sides

Then you get x=27.6

Please help me with this on the image

Answers

Answer:

hope this may help u

q. no (a)= 60%

(b)=55%

Answer:

a) [tex]\frac{6}{10}[/tex] = [tex]\frac{3}{5}[/tex]  = .6 or 60%

b) [tex]\frac{11}{20}[/tex]  = .55 or 55%

Step-by-step explanation:

Count the boxes.

[tex]\frac{shaded}{total}[/tex]

Divide and multiply by 100

6 – x + = 6 minus StartFraction 3 Over 4 EndFraction x plus StartFraction 1 Over 3 EndFraction equals StartFraction one-half EndFraction x plus 5.X + 5 2 3 6 12

Answers

Answer:

x = 16/15

Step-by-step explanation:

Given:

6 - 3/4x + 1/3 = 1/2x + 5

Collect like terms

6 + 1/3 - 5 = 1/2x + 3/4x

(18+1-15)/3 = (2x+3x)/4

4/3 = 5/4x

x = 4/3 ÷ 5/4

x = 4/3 × 4/5

x = (4 * 4) / (3 * 5)

x = 16/15

(Hurry giving brainliest) Which idea does Arnold need to correct? In specular reflection, reflected rays move in the same direction. In scattering, light strikes particles in the atmosphere. In diffuse reflection, reflected rays move in different directions. In scattering, long wavelengths are scattered the most.

Answers

Answer:

D. In scattering, long wavelengths are scattered the most.

Step-by-step explanation:

Scattering of light is the effect observed when there is an interaction between light energy and some particles. This occurs often in the earth's atmosphere which consists of different sizes of particles that interacts with the sunlight.

In scattering, light of shorter wavelengths are scattered by small sized particles compared to those with longer wavelengths. Example is the scattering of the sunlight by the atmospheric particles where colors with short wavelengths are scattered the most.

Arnold need to correct the statement; In scattering, long wavelengths are scattered the most. To read; In scattering, short wavelengths are scattered the most.

Answer:

D on edgen

Step-by-step explanation:

What is the measure of ∠
A. 6°

B. 42°

C. 60°

D. 49°

Answers

Answer:

<XYZ is equal to 49°

Step-by-step explanation:

Set the two expressions equal to each other so 7x+7=5x+19. Subtract 5x from 7x and 7 from 19 which is equal to 2x=12 that means x is 6. then plug 6 into (7x+7) which is equal to 49.

Oscar bought 15 gallons of water at $1.98 per gallon. He wants to divide this water in bottles of 1/8 gallon each. What is the cost of a bottle of water?

Answers

Answer:

Step-by-step explanation:

Other Questions
Sarah is often anxious and sad about a variety of things in life. When she perceives that she is wronged by someone, she has a tendency to react harshly on that person. Sarah feels generally dissatisfied with the way her life is going. Her personality is characterized by ____. hi guys, which one do find correct =she has finally gotten herself a job;she has finally found a job;or she finally found a job;she finally got herself a job 3. Which term describes the offspring of afirst-generation cross between parents withdifferent forms of a trait? sc.7.L.16.1 Which expression is equivalent to ____? You (receive) any letters from your parents yet ?How long Bod and Mary (be) married ?My father ( not smoke) for 5 yearsBirds ( build ) there nests in the summer and (fly ) to the south in winterIm busy at the moment. I ( redecorate ) the sitting room By selling a mobile for Rs.30, 000, Shankar gains 5%. Find the CP of the mobile. An isosceles right triangle has a hypotenuse that measures 42 cm. What is the area of the triangle?PLEASE HELP A money market security that has a par value of $10,000 sells for $8,924.70. Given that the security has a maturity of two years, what is the investor's required rate of return? XYZ Co. uses a job-order costing system with a single plantwide predetermined overhead rate based on machine-hours. It based its predetermined overhead rate for the current year on total fixed manufacturing overhead cost of $440,000, variable manufacturing overhead of $2.20 per machine-hour, and 50,000 machine-hours. The estimated total manufacturing overhead is closest to: Calculate the minimum area moment of inertia for a rectangular cross-section with side lengths 6 cm and 4 cm.52 cm472 cm432 cm424 cm42 cm4 Solve the equation by completing the square.0 = 4x2 72x An equation is shown below:3(4x 2) = 1Which of the following correctly shows the steps to solve this equation? Step 1: 12x 2 = 1; Step 2: 12x = 3 Step 1: 12x 6 = 1; Step 2: 12x = 7 Step 1: 7x + 1 = 1; Step 2: 7x = 0 Step 1: 7x 5 = 1; Step 2: 7x = 6 Which of these words is an irregular verb?A. switchB. shakeC. yellD. toggle "The most important issue going forward is how AI technology can best serve the humans who created it."How does this sentence engage the reader in the subject material?A. The author draws a conclusion that the reader is expected to accept.B. The author uses humor to encourage the reader to accept the information.C. The author invites the reader to consider the possibilities of this technology.D. The author tells the reader that this technology will improve his or her life. Use your understanding of the political philosophies Founding Fathers. Which phrase shows that Patrick Henry was anti-federalist in his ideas?"The Constitution is not an instrument for the government to restrain the people; it is an instrument for the people to restrain the government- lest it come to dominate our lives and interests."-- Patrick Henry-It is an instrument for the people to restrain the government This phrase talks about literally restraining the government and giving power to the people which is what he believed in. This statement is bold and straight up. While out for a run, two joggers with an average age of 55 are joined by a group of three more joggers with an average age of m. if the average age of the group of five joggers is 45, which of the following must be true about the average age of the group of 3 joggers?a) m=31b) m>43c) m please help me answer for this quickly what is the charge on ion X Li2X the distance between (-4,-5) and (4,2) A tall plant is crossed with a short plant, and the progeny are all intermediate in size between the two parental plants. If the intermediate F1 progeny were allowed to self-pollinate, and the F2 progeny were also intermediate in size, but following a normal distribution, this would suggest